3
$\begingroup$

Let $n\geq 2$. Set $\Sigma= \{x\in \mathbb{R}^n: 1\leq |x|<2\}$. Assume $h:\Sigma \rightarrow \mathbb{R}^n$ is continuous and injective.

Question: Must $h$ also be an embedding?

Some thoughts:

  1. $h|_{\operatorname{int}\Sigma}$ is an embedding.

  2. $h(\operatorname{int}\Sigma)$ is open.

  3. If $h$ defined on $\overline{\Sigma}$ then YES.

  4. If $\dim \Sigma <n$ then NO. Consider $\alpha:[0,1)\rightarrow \mathbb{R}^2$, $\alpha(t)=(\cos(2\pi t),\sin(2\pi t))$.

  5. The map $h|_{\mathbb{S}^{n-1}}$ is an embedding.

Observe $h(\Sigma)=h(\operatorname{int} \Sigma)\coprod h(\mathbb{S}^{n-1})$.

$\endgroup$
3
  • $\begingroup$ I might be misunderstanding something here, since I don't see how 6 is true, but it seems false in dimension $1$, since $\Sigma$ is disconnected but you can arrange for $h$ to have connected image. $\endgroup$ Sep 7 at 15:03
  • $\begingroup$ @AlessandroCodenotti Sorry yes I was thinking of higher dimensions, and disregard $6$. $\endgroup$ Sep 7 at 15:09
  • 1
    $\begingroup$ I guess yes, but not sure. Since $\Sigma = [0;1)\times S^{n-1}$ and $h\big|_{S^{n-1}\times[0;1/2]}$ is an embedding, it suffices to prove that in some neighbourhood of $h(S^{n-1}\times\{0\})$ there are no points of $h(S^{n-1}\times(1/2;1)).$ By Alexander duality $h(S^{n-1}\times\{0\})$ splits the space into two parts, one of which is already locally occupied by $h(S^{n-1}\times(0;1/2)).$ Since $h$ is injection and since $h(S^{n-1}\times(0;1))$ lies to the one side of $h(S^{n-1}\times\{0\})$, the claim follows. $\endgroup$ Sep 8 at 11:41

1 Answer 1

2
$\begingroup$

The answer is yes. It suffices to show that $h$ restricted to each of the sets $1\leq |x|\leq 3/2$ and $3/2\leq |x|<2$ is an embedding, because the image of points near the boundary $|x|=2$ will be away from the image of $1\leq |x|\leq 3/2$. Indeed, by the generalized Jordan-Brouwer separation theorem $1\leq |x|\leq 3/2$ and $3/2\leq |x|<2$ will be in different components of $\mathbb{R}^n\setminus f(\{|x|=3/2\})$.

Thus the only problem is to verify that $h$ is an embedding near $|x|=2$. However, according to The generalized Schoenflies theorem of Mazur and Brown the map $h$ restricted to $3/2\leq |x|<2$ extends to an embedding of $|x|<2$ and hence $h$ is an embedding of $3/2\leq |x|<2$.

Here is a second (even more elementary) argument:

According to Brouwer's invariance of domain theorem $h$ restricted to $1<|x|<2$ is an embedding. It is also an embedding of $1\leq |x|\leq 3/2$ (by compactness of the domain). Since the image of $|x|=1$ is at a positive distance to the image of $3/2\leq |x|<2$ (by the Jordan-Brouwer separation theorem as explained above), it follows that $h$ is an embedding of $1\leq |x|<2$.

$\endgroup$
0

Your Answer

By clicking “Post Your Answer”, you agree to our terms of service and acknowledge that you have read and understand our privacy policy and code of conduct.

Not the answer you're looking for? Browse other questions tagged or ask your own question.